18. The weights of four puppies are shown in pounds. 9.5 9 9.125 9 Which list shows these weights in order from greatest to least F. 99.5 9 9.125 w 9.5 9 9.125 9.125 9 9.5 9 + J. 9 9 9.5 9.125

18. The Weights Of Four Puppies Are Shown In Pounds. 9.5 9 9.125 9 Which List Shows These Weights In

Answers

Answer 1

The correct list is

[tex]9\frac{3}{4},\text{ 9.5, 9}\frac{3}{8},9.125[/tex]

This is option F


Related Questions

two integers a and b have a product of 36 what is the least possible answer

Answers

Answer:

the answer is 12

Step-by-step explanation:

In short, a = 6 and b = 6. We can make a table of various values to help confirm that 12 is the smallest sum.

Answer: 1

Step-by-step explanation: 1 is the least possible answer because an integer is any number and 1x36 would be the least possible answer for the product.

Which of the following graphs is a polynomial function with intercepts of(-2,0), (1, 0), and (4, 0)711-15 4NO C.O D.

Answers

Explanation

We are given the following:

We are required to determine which of the following graphs is a polynomial function with intercepts of

(-2,0), (1, 0), and (4, 0).

This can be achieved by looking for the graph that crosses the x-axis at the points -2, 1 and 4.

Hence, the answer is option C.

Mrs. Smith has 12 times as many markers as colored pencils. The total number of markers and colored pencils is 78. How many markers does Mrs. Smith have?ok...answers given so far are not helpful in explaining process.

Answers

Let:

x = Colored markers

y =

You pick a card at random.
1 2 3 4
What is P(factor of 24)?
Write your answer as a percentage rounded to the nearest tenth

Answers

Answer:

100%

Step-by-step explanation:

All of the numbers are factors of 24. So, picking a factor of 24 is guaranteed, so the probability is 1.

This is equal to 100%.

Jackson deposited $160 a month into an account earning 7.2% compounded monthly for 12 years. He left the accumulated amount for another 3 years at the same interest rate. How much total interest did he earn?

Answers

Using the formula for the compound interest, we have:

[tex]\begin{gathered} A=P(1+\frac{r}{n})^{nt} \\ \text{ A:amount,P:principal,r:rate,n: number of times interest is compounded per year,t:time in years.} \\ A=160(1+\frac{0.072}{12})^{12\cdot15}\text{ (Replacing the values)} \\ A=160(1+0.006)^{180}\text{ (Dividing and multiplying)} \\ A=160\cdot2.935\text{ (Adding and raising the result to the power of 180)} \\ A=469.63\text{ (Multiplying)} \\ \text{Interest}=\text{ Amount - Principal=469.63}-160=309.63 \\ \text{The answer is \$309.63} \end{gathered}[/tex]

Which of the following is a solution to the inequality below?

Answers

Answer:

4u + 6 > 30

4u > 24

u > 6

Solution is u = 6

Answer:

4u+6>30

4u>30-6

4u>24

u>6

Answer:

u>6

please help this is for my study guide thanks! (find volume) (don't round)

Answers

100,000π ft³

1) Let's find the volume of that Cylinder using this formula:

[tex]V=\pi r^2h[/tex]

Note that the volume is the area of the base (a circle) times the height

2) Also, notice that in the picture we have the diameter, the radius is half the Diameter:

[tex]\begin{gathered} V=\pi\cdot(50)^2\cdot40 \\ V=100000\pi^{} \end{gathered}[/tex]

3) So the volume is 100,000π ft³

Question 3 4.5 pts At the honor roll party, students had the choice of cheese or pepperoni pizza and coke or sprite. Of the 125 students that made the honor roll 64% had cheese pizza. There were 48 students that had cheese pizza and a coke. 5 more students chose to have a Coke rather than Sprite. Complete the table below.

Answers

The table would look like this;

We are told that Of the 125 students that made the honor roll 64% had cheese pizza.

64% of 125 is 80 students, therefore, 80 students in total had cheese pizza.

Let's fill that in.

We now know that those who had pepperoni pizza are 125 - 80 = 45 in number.

There were 48 students that had cheese pizza and a coke, let's fill that in too, we have.

This means that the number of students that had a cheese and sprite is 80 - 48 = 32 students.

We are also told that 5 more students chose to have a coke than a sprite.

Let the total number that chose coke be x.

Then the total who chose sprite would be x - 5.

But these total must add up to 125.

So;

[tex]\begin{gathered} x+x-5=125 \\ 2x-5=125 \\ 2x=130 \\ x=\frac{130}{2}=65 \\ x-5\text{ = 60} \end{gathered}[/tex]

Therefore, 65 students took coke in total and 60 took sprite, let's fill that in too.

We can now fill in the pepperoni column.

For pepperoni and coke, we subtract 48 from 65 to obtain 17

For pepperoni and sprite, we subtract 32 from 60 to obtain 28

ii. The joint relative frequency of the students who had a sprite and pepperoni pizza.

From the table, the joint relative frequency of those who had a sprite and a pepperoni pizza is

[tex]\begin{gathered} \frac{28}{45} \\ \end{gathered}[/tex]

i.e 28/45 or 0.6 of those who had pepperoni pizza, took sprite.

When a scientist conducted a genetics experiments with peas, one sample of offspring consisted of 953 peas, with 746 of them having red flowers. If we assume, as the scientist did,
that under these circumstances, there is a 3 / 4 probability that a pea will have a red flower, we would expect that 714.75 (or about 715) of the peas would have red flowers, so the result
of 746 peas with red flowers is more than expected.
a. If the scientist's assumed probability is correct, find the probability of getting 746 or more peas with red flowers.
b. Is 746 peas with red flowers significantly high?
c. What do these results suggest about the scientist's assumption that 3 / 4 of peas will have red flowers?
a. If the scientist's assumed probability is correct, the probability of getting 746 or more peas with red flowers is

Answers

a. 74.99% probability of getting 746 or more peas with red flowers.

b. Since Z < 2, 746 peas with red flowers is not significantly high.

c. Since 746 peas with red flowers is not a significantly high result, we cannot conclude that the scientist's assumption is wrong.

Given,

953 peas in sample with 746 of them having red flower

Scientist's assumption;

Since there is a 3/4 chance that a pea will have a red blossom, we would anticipate 714.75 (or roughly 715) of the peas to do so; hence, the finding of 746 peas with red flowers is higher than we had anticipated.

Here,

Binomial distribution:

Probability of x successes on n trials, with p probability.

Normal distribution:

In a normal distribution with mean μ and standard deviation σ, the z-score of a measure X is given by:

Z = (X - μ) / σ

If  np ≥ 10 and n (1 - p) ≥ 10 , the binomial distribution can be approximated to the normal with:

μ = np

σ = [tex]\sqrt{np(1-p)}[/tex]

Here,

n = 953 and p = 3/4 = 0.75

Lets see,

μ = np = 953 x 0.75 = 714.75

σ = [tex]\sqrt{np(1-p)}[/tex] = [tex]\sqrt{714.75 . (1 - 0.75)}[/tex] = √714.5 = 26.73

a. The probability of getting 746 or more peas with red flowers.

Using continuity correction, this probability is P(X ≥ 746 - 0.5) = P(X ≥ 745.5) , which is 1 subtracted by the p-value of Z when X = 745.5.

Then:

Z = (X - μ) / σ = (745.5 - 714.75) / 26.73 = 30.75 / 26.73 = 1.150

The p value of z score 1.150 is 0.2501

1 -0.2501 = 0.7499

0.7499 = 74.99% probability of getting 746 or more peas with red flowers.

b. Is 746 red-flowering peas noticeably high

Since Z < 2, 746 peas with red flowers is not significantly high.

c. What do these findings say about the researcher's prediction that 3/4  pea plants will have red flowers?

Since 746 peas with red flowers is not a significantly high result, we cannot conclude that the scientist's assumption is wrong.

Learn more about normal distribution here;

https://brainly.com/question/25082768

#SPJ1

I need help with finding the output y when x is -4 it's on a graph

Answers

As observed from the graph, the curve is a straight line from point (-2,-1) to (-5,2).

Consider that the equation of a straight line passing through two points is given by,

[tex]y-y_1=\frac{y_2-y_1}{x_2-x_1}\times(x-x_1)[/tex]

So the equation of the line passing through (-2,-1) and (-5,2) is given by,

[tex]\begin{gathered} y-(-1)=\frac{2-(-1)}{-5-(-2)}\times(x-(-2)) \\ y+1=\frac{3}{-3}\times(x+2) \\ y+1=-x-2 \\ y=-x-3 \end{gathered}[/tex]

Note that this function is only for the interval [-2, -5].

Now, the value of 'y' corresponding to the input x=-4 is calculated as,

[tex]\begin{gathered} y=-(-4)-3 \\ y=4-3 \\ y=1 \end{gathered}[/tex]

Thus, the required output is y = 1 .

Find the Value of interval [0,2pie] such as that tan s= -radical3/3

Answers

The values of s in the interval [0, 2π) such that tan s = -(√3)/3 are 5π/6 and 11π/6.

What is trigonometry and how is it assessed?
Simply put, trigonometric functions—also referred to as circular functions: are the functions of a triangle's angle. This means that these trig functions provide the relationship between the angles and sides of a triangle. Sine, cosine, tangent, cotangent, secant, and cosecant are the fundamental trigonometric functions. Numerous trigonometric identities and formulas indicate the relationship between the functions and aid in determining the triangle's angles.
The quadrants determine the values of the trigonometric functions.


Given, tan s = -(√3)/3 ⇒ tan s = -(√3)/(√3)² ⇒ tan s = (-1)/(√3)
Therefore, the simplified value of tangent of s is tan s = (-1)/(√3)
Again the interval of the function is [0, 2π), so only the second and fourth quadrants can contain the given value of tangent being negative.
For the value of s in second quadrant, we have:
tan s = (-1)/(√3) ⇒ tan s = tan (π - (π/6)) ⇒ tan s = tan (5π/6) ⇒ s = 5π/6
For the value of s in the fourth quadrant, we have:
tan s = (-1)/(√3) ⇒ tan s = tan (2π - (π/6)) ⇒ tan s = tan (11π/6) ⇒ s = 11π/6
Thus, the values of s in the interval [0, 2π) such that tan s = -(√3)/3 are 5π/6 and 11π/6.

To learn more about trigonometry, tap on the link below:
https://brainly.com/question/24349828

#SPJ13

Which pair of numbers are not opposites?
47 and- 47
74 and -74
|4| and -4
47 and |-47|

Answers

FOR THE PAIRS TO BE OPPOSITE IT MEANS THE NUMBERS SHOULD ALSO CONTRAST IN SIGNS.

47 AND -47 ARE OPPOSITE

74 AND - 74 ARE OPPOSITE

|4|=4 AND -4 ARR OPPOSITE

47 AND |-47|=47 ARE NOT OPPOSITE BECAUSE THEY BOTH HAVE THE SAME SIGNS.

THE LAST OPTION IS THE ANSWER.

There are 45 boys and 81 girls in a dance competition. What is the ratio of boys to girls, in the simplest form?

Answers

Answer

[tex]\frac{5}{9}[/tex]

Explanation

Given

• 45 boys

,

• 81 girls

Procedure

We have to find the ratio of boys to girls, which can be written as 45:81 or:

[tex]\frac{45}{81}[/tex]

However, we have to simplify. Both numbers are multiple of 9, thus:

[tex]=\frac{\frac{45}{9}}{\frac{81}{9}}=\frac{5}{9}[/tex]

Then every five boys there are 9 girls.

What is the solution to x^2 – 9x < –8?A. x < 1 or x > 8B. x < –8 or x > 1C. 1 < x < 8D. –8 < x < 1

Answers

INFORMATION:

We have the next inequality

[tex]x^2-9x<-8[/tex]

And we must find its solution

STEP BY STEP EXPLANATION:

To solve it, we must:

1. Move all terms aside

[tex]x^2-9x+8<0[/tex]

2. Factor x^2-9x+8

[tex](x-8)(x-1)<0[/tex]

3. Solve for x

[tex]x=8\text{ or }x=1[/tex]

4. From the values of x, we have these 3 intervals to test

[tex]\begin{gathered} x<1 \\ 18 \end{gathered}[/tex]

5. Choose a test point for each interval

For the interval x < 1:

[tex]\begin{gathered} \text{ Using x }=0, \\ 0^2-9(0)<-8 \\ 0<-8 \end{gathered}[/tex]

which is false. So, the interval is discarded.

For the interval 1 < x < 8:

[tex]\begin{gathered} \text{ Using x }=2, \\ 2^2-9(2)<-8 \\ -14<-8 \end{gathered}[/tex]

which is true. So, the interval is maintained

For the interval x > 8:

[tex]\begin{gathered} \text{ Using x = 9,} \\ 9^2-9(9)<-8 \\ 0<-8 \end{gathered}[/tex]

which is false. So, the interval is discarded.

Finally, the solution would be the interval that was maintained: 1 < x < 8.

ANSWER:

C. 1 < x < 8

Answer:

C. 1 < x < 8

Step-by-step explanation:

x² - 9x < -8

we will suppose some values for x to check which values will satisfy this inequality:

for x = 1

1(1-9) < -8 which is wrong

for x = 2

2(2-9) < -8 this is satisfying the inequality

for x = 8

8(8-9) < -8 which is wrong

let's take any negative value now,

let x = -2

-2(-2-9) < -8 which is wrong

thus x is the positive value which will always be greater than 1 and less than 8 for the given inequality.

A parabola contains the following points.(-5,8),(2,-3),(3,1) which of the following systems of equations could be solved in order to find the equation that corresponds to this parabola?

Answers

Generic parabola equation:

y = a*x^2 + b*x + c

We have three points of the parabola:

(-5,8), (2,-3), (3,1)

For the point (-5, 8): x = -5, y = 8

8 = 25*a - 5*b + c

Point (2,-3): x = 2, y = -3

-3 = 4*a + 2*b + c

Point (3, 1): x = 3, y = 1

1 = 9*a + 3*b + c

Our system of equations:

8 = 25*a - 5*b + c

-3 = 4*a + 2*b + c

1 = 9*a + 3*b + c

The last option is the correct answer

May someone please help me solve this and explain? thanks:)

Answers

Given:

Mean,

[tex]\mu=46[/tex]

Standard deviation,

[tex]\sigma=7[/tex]

To find: The indicated values

Explanation:

The values are calculated as follows,

[tex]\begin{gathered} \mu-3\sigma=46-3(7) \\ =46-21 \\ =25 \\ \mu-2\sigma=46-2(7) \\ =46-14 \\ =32 \\ \mu-\sigma=46-7 \\ =39 \\ \mu=46 \\ \mu+\sigma=46+7 \\ =53 \\ \mu+2\sigma=46+2(7) \\ =46+14 \\ =60 \\ \mu+3\sigma=46+3(7) \\ =46+21 \\ =67 \end{gathered}[/tex]

Final answer: The values are,

[tex]\begin{gathered} \mu-3\sigma=25 \\ \mu-2\sigma=32 \\ \mu-\sigma=39 \\ \mu=46 \\ \mu+\sigma=53 \\ \mu+2\sigma=60 \\ \mu+3\sigma=67 \end{gathered}[/tex]

40% of what number is 26? Please show work!

Answers

65

1) To find that, we need to write an equation:

[tex]x(0.4)=26[/tex]

Note that we rewrote that 40% as 0.4.

2) Now, let's solve it

[tex]\begin{gathered} x0.4=26 \\ \frac{0.4x}{0.4}=\frac{26}{0.4} \\ x=65 \end{gathered}[/tex]

3) So the 26 is 40% of 65

AB is a median of a triangle true or false

Answers

To answer this question, first we need to understand the definition of a median of a triangle.

A median of a triangle is a line segment drawn from a vertex to the midpoint of the opposite side of the vertex.

AB is a segment drawn from the vertex A, to the point B, but B is not the midpoint of the base of this triangle(the midpoint divides the segment into two equal parts, and since one part is 7 and the other is 8, B is not the midpoint

determine if the following equations represent a linear function if so write it in standard form Ax+By=C9x+5y=102y+4=6x

Answers

9x + 5y = 10

is a linear equation because all variables are raised to exponent 1.

This equation is already written in standard form (A = 9, B = 5, C = 10)

2y + 4 = 6x​

is a linear equation because all variables are raised to exponent 1.

Subtracting 2y at both sides:

2y + 4 - 2y= 6x​ - 2y

4 = 6x​ - 2y

or

6x - 2y = 4

which is in standard form (A = 6, B = -2, C = 4)

gB - N³B = d what does B equal?

Answers

Answer:

[tex]b \: = \frac{d}{(g - {n}^{3} )} [/tex]

Hi, can you help me to solve this problem, please!!

Answers

In this problem, we have a vertical parabola open downward

that means

the vertex represents a maximum

looking at the graph

the maximum has coordinates (1,9)

therefore

the vertex is (1,9)

let f ( x ) = 6356 x + 5095 . Use interval notation. Many answers are possible.

Answers

The equation of the function has its domain representation in interval notation as (oo, oo)

How to determine the domain of the function

From the question, the equation of the function is given as

f ( x ) = 6356 x + 5095

Rewrite the equation of the function properly by removing the excess spaces

So, we have

f(x) = 6356x + 5095

The above equation is a linear equation

A linear equation is represented as

f(x) = mx + c

As a general rule;

The domain of a linear equation is all set of real numbers

This is the same for the range

i.e. the range of a linear equation is all set of real numbers

When the set of real numbers is represented as an interval notation, we have the following representation

(oo, oo)

Hence, the domain is (oo, oo)

Read more about domain at

https://brainly.com/question/2264373

#SPJ1

Possible question

let f ( x ) = 6356 x + 5095 . Use interval notation to represent the domain of the function.

Many answers are possible.

find the perimeter of the triangle whose vertices are (-10,-3), (2,-3), and (2,2). write the exact answer. do not round.

Answers

We have to calculate the perimeter of a triangle of which we know the vertices.

The perimeter is the sum of the length of the three sides, which can be calculated as the distance between the vertices.

The vertices are V1=(-10,-3), V2=(2,-3), and V3=(2,2).

We then calculate the distance between each of the vertices.

We start with V1 and V2:

[tex]\begin{gathered} d_{12}=\sqrt[]{(y_2-y_1)^2+(x_2-x_1)^2} \\ d_{12}=\sqrt[]{(-3-(-3))^2+(2-(-10)^2} \\ d_{12}=\sqrt[]{(-3+3)^2+(2+10)^2} \\ d_{12}=\sqrt[]{0^2+12^2} \\ d_{12}=12 \end{gathered}[/tex]

We know calculate the distance between V1 and V3:

[tex]\begin{gathered} d_{13}=\sqrt[]{(y_3-y_1)^2+(x_3-x_1)^2} \\ d_{13}=\sqrt[]{(2-(-3))^2+(2-(-10))^2} \\ d_{13}=\sqrt[]{5^2+12^2} \\ d_{13}=\sqrt[]{25+144} \\ d_{13}=\sqrt[]{169} \\ d_{13}=13 \end{gathered}[/tex]

Finally, we calculate the distance between V1 and V3:

[tex]\begin{gathered} d_{23}=\sqrt[]{(y_3-y_2)^2+(x_3-x_2)^2} \\ d_{23}=\sqrt[]{(2-(-3))^2+(2-2)^2} \\ d_{23}=\sqrt[]{5^2+0^2} \\ d_{23}=5 \end{gathered}[/tex]

Then, the perimeter can be calcualted as:

[tex]\begin{gathered} P=d_{12}+d_{13}+d_{23} \\ P=12+13+5 \\ P=30 \end{gathered}[/tex]

Answer: the perimeter is 30 units.

if 2 angles from a line

Answers

If two angles form a linear pair, then they form a straight line, and the sum of their measures is 180 degrees.

This illustrated below;

In the illustration above, angle measure 1 and 2 both equal to 180 degrees. Angle 1 and angle 2 are refered to as a linear pair.

for each triangle list the sides in order from shortest to longest explain your reasoning with words or numbers for the order

Answers

a. For the first triangle ,

Two angles are given. To determine the third angle apply the property of triangle which is sum of the angles of the triangle is 180 degree.

[tex]27^{\circ}+82^{\circ}+\angle T=180^{\circ}[/tex][tex]\angle T=180^{\circ}-27^{\circ}-82^{\circ}=71^{\circ}[/tex]

The triangle angles and sides relationship-

The longest side of a triangle is opposite the biggest angle measure.

The shortest side of a triangle is opposite the smallest angle measure in a triangle.

Therefore,

The shortest side is side opposite to the smallest angle that is MT.

The largest side is side opposite to the largest angle that is AT.

Hence the order for the sides from shortest to largest is

[tex]MT

b. The triangle is given . First determine the value of x and find the angle using the property of triangle which is sum of the angles of the triangle is 180 degree.

[tex]8x-1+3x+4+3x+9=180^{\circ}[/tex][tex]14x+12=180[/tex][tex]14x=168[/tex][tex]x=12[/tex]

The angles obtained are

[tex](8x-1)=95,(3x+4)=40,(3x+9)=45[/tex]

We know that the longest side of a triangle is opposite the biggest angle measure.

The shortest side of a triangle is opposite the smallest angle measure in a triangle.

Hence the shortest side is JK and largest side is JL.

Hence the order for the sides from shortest to largest is

[tex]JK

triangle HXI can be mapped onto troangle PSL by a reflection If m angle H = 157 find m angle S

Answers

From the information provided, the triangle HXI can be mapped onto triangle PSL. This means the vertices of the reflected image would now have the following as same measure angles;

[tex]\begin{gathered} \angle H\cong\angle P \\ \angle X\cong\angle S \\ \angle I\cong\angle L \end{gathered}[/tex]

Measure of angle S cannot be determined from the information provided because there is insufficient information given to determine the measure of angle X, hence the angle congruent to it (angle S) likewise cannot be determined.

Leah invested $400 in an account paying an interest rate of 1 1/2%compounded annually. Lauren invested $400 in an account paying aninterest rate of 0 7/8% compounded monthly. To the nearest hundredth of ayear, how much longer would it take for Lauren's money to triple than forLeah's money to triple?

Answers

Leah investment is:

[tex]M_{\text{Leah}}=400_{}\cdot1.5^y[/tex]

Where M is the ammount of money that she has, and y the number of years.

We want to know the number of years that must elapse for her investment to triple, so we want to know the value of y such that:

[tex]\begin{gathered} 3\cdot400=400\cdot(1+\frac{1.5}{100})^y \\ 3=(1.015)^y \\ \ln 3=y\cdot\ln (1.015) \\ y=\frac{\ln (3)}{\ln (1.015)}\cong73.788\cong73.79 \end{gathered}[/tex]

It will take 73.79 years to triple her investment.

Lauren investment is:

[tex]M_{\text{Lauren}}=400\cdot(1+\frac{7}{8}\cdot\frac{1}{100})^m=400\cdot(1.00875)^{\frac{y}{12}}[/tex]

Where M is the ammount of money that she has, and m the number of months, and y is the number of years.

We want to know the number of years that must elapse for her investment to triple, so we want to know the value of y such that:

[tex]\begin{gathered} 3\cdot400=400\cdot(1.00875)^{\frac{y}{12}} \\ 3=(1.00875)^{\frac{y}{12}} \\ \ln 3=\frac{y}{12}\ln (1.00875) \\ y=12\cdot\frac{\ln 3}{\ln (1.00875)} \\ y=1513.25 \end{gathered}[/tex]

I think is the average of the highest point and the lowest one, what's the midline of the graph?

Answers

The Midline of a Sinusoid

A sinusoid is a periodic function which parent expression is:

f(x) = A. sin (wt)

Where A is the amplitude and w is the angular frequency

The sine function has a maximum value of A and a minimum value of -A.

The midline can be found as the average value of the maximum and the minimum value.

For the parent function explained above, the midline is:

[tex]M=\frac{\text{Mx}+Mn}{2}[/tex]

Since Mx and Mn are, respectively A and -A, the midline is zero.

The graph shown in the image has a maximum of Mx=1 and a minimum of Mn=-5.

Thus, the midline is:

[tex]M=\frac{\text{1}-5}{2}=-\frac{4}{2}=-2[/tex]

The midline of the graph is y=-2

Compute the derivative of f(x) = x * sin(x) . Use the result to compute f^ prime ( pi 2 )

Answers

Solution

Step 1

f(x) = xsinx

Step 2

[tex]\begin{gathered} f(x)=x\sin\left(x\right) \\ \\ f^{\prime}(x)=\sin\left(x\right)+x\cos\left(x\right) \end{gathered}[/tex]

Step 3

[tex]\begin{gathered} f^{\prime}(\frac{\pi}{2})=\sin\left(\frac{\pi}{2}\right)+\frac{\pi}{2}\cos\left(\frac{\pi}{2}\right) \\ \\ f^{\prime}(\frac{\pi}{2})=1 \end{gathered}[/tex]

Final answer

A. 1

Ninety percent of a large field is cleared for planting. Of the cleared land, 50 percent is planted with blueberry plants and 40 percent is planted with strawberry plants. If the remaining 360 acres of cleared land is planted with gooseberry plants, what is the size, in acres, of the original field?*

Answers

For the given question, let the size of the original field = x

Ninety percent of a large field is cleared for planting

So, the size of the cleared land = 90% of x = 0.9x

50 percent is planted with blueberry plants and 40 percent is planted with strawberry plants.

So, the size of the land planted with blueberry plants and strawberry plants =

[tex]0.5\cdot0.9x+0.4\cdot0.9x=0.45x+0.36x=0.81x[/tex]

The remaining will be = 0.9x - 0.81x = 0.09x

Given: the remaining 360 acres of cleared land is planted with gooseberry plants

so,

[tex]0.09x=360[/tex]

divide both sides by (0.09) to find x:

[tex]x=\frac{360}{0.09}=4,000[/tex]

So, the answer will be:

The size of the original field = 4,000 acres

Other Questions
I ONLY HAVE 50 MINUTES PLEASE HELPOne major difference between accountability standards for social workers compared with addiction recovery counselors is that social workers must be certified and licensed to practice, whereas addiction recovery counselors require only _____.a. permissionb. associate degreec. certificationd. a GED n the diagram, the circle will be dilated by a scale factor of 3 about the origin. The points C, A, and B map to C', A', and B' after the dilation. What is the length of ? Use the distance formula to help you decide: . The graph shows a circle with three points, A is equal to (8, 15), B is equal to (12, 13), and C is equal to (8, 10). C is the center of the circle. A line is joining AC and BC A. 24 units B. 21 units C. 45 units D. 5 units E. 15 units the dean of a college faces the following costs: graders, faculty, classroom space, and chalk. of these costs, which are likely to be variable in the long run? I need help with my pre-calculus homework, please show me how to solve them step by step if possible. The image of the problem is attached. These are 2 parts of the same question. The line parallel to y=3x+5 that passes through the point (-8,-4) Find the measure of the indicated angle to the nearest degree.A. 63B. 25C. 31D. 27 bowzer company has just received $2.8 million from the sale of one of its divisions. the company has 380,000 shares outstanding that sell for $84.27 per share. if the company issues the entire proceeds from the sale as a special dividend, what will the ex-dividend stock price be? ignore taxes. = The number of counties in state A and the number of counties in state B are consecutive even integers whose sum is 82. If state A has more counties than state B, how many counties does each state have? State A has counties. A class has 21 children, 10 are girls and 11 are boys. what fraction of the class is made up of boys? (The question is in the photo Sorry Lol) What volume of chlorine gas at 36.0C and 1.74 atm is needed to react completely with12.0 g of sodium to form NaCl?Answer in units of L The sum of two numbers is ten. One number istwenty less than four times the other. Find thenumbers.Note: List numbers with a comma separatingthem, e.g. 5, 12. which statement is true and why? & why not the others? Using f(x) = 2x - 3 and g(x) = 5, find f(g(3)).7530None of the choices are correct.I dont think my answer is right please help me thank you b. Find a pair of numbers that have a sum of 50 and will produce the largest possible product. Example: +_ = 50 (sum) so _* _ = _ (maximum area) and (enter answers from the sum) You pay $0. 25 per name for list a and get a click rate of 5%. You pay $0. 35 per name for list b and get a click rate of 10%. Which list has the lower cost-per-click?. What is the importance in Et Tu Brute?- Then fall, Caesar! and who said it? Find f(x) g(x) if f(x) = x2 7 and g(x) = x2 + 3x + 7 please help me with this question y varies directly as x, y = 7 when x = 21. Determine x when y = 5. the purpose of title insurance is to: multiple choice protect an owner from changes in mortgage interest rates. transfer ownership of property. protect an owner from unforeseen property claims. cover the closing costs of a real estate transaction. reduce the chance of a decrease in property value.